Solve the equation.
4.5=0.5n

Answers

Answer 1
Divide by 0.5 on both sides
4.5 divided by 0.5 = 9
Have a nice day

Related Questions

what is the answer in system form
y=-2x
5x-7y=-38

Answers

Answer:

x = -2

y = 4

Step-by-step explanation:

To solve the system of equations, you want to plug one equation into the other and then simplify. You can do this by setting one equation equal to a variable (like in the first equation), and then substituting the function into the variable into the second equation.

First Equation: y = -2x

Second Equation: 5x - 7y = -38

5x - 7y = -38                                <----- Second equation

5x - 7(-2x) = -38                           <----- Plug first equation into "y"

5x + 14x = -38                              <----- Multiply -7 and -2x

19x = -38                                      <----- Add 5x and 14x

x = -2                                           <----- Divide both sides by 19

Now that you know the value of one variable, you can use it to find the value of the second variable. This can be done by plugging x = -2 in to one of the equations.

y = -2x                                         <----- First equation

y = -2(-2)                                      <----- Plug -2 in "x"

y = 4                                            <----- Multiply -2 and -2

C
2
1
3
A. 21
B. 23
C.24
D. 25
b7
#
5
4
In the figure, which angle has the same measure as /2?

Answers

Answer:

the angle that has the same measure as /2 is A

I think its 2c but am not sure

Brandon is a running back for his local high school football team. In the last game, he carried the ball 8 times. In the first 5 carries, he gained 6 yards, 12 yards, and 4 yards before losing 2 yards and then losing additional yards. His last 7 carries combined for yards. What was his total net yardage for the game?

Answers

Using it's concept, it is found that Brandon's net yardage for the game was of 26.

How to find the net yardage?

The total net yardage is the sum of all the yardage they gain, that is, the positive gains are added with a plus signal, while the negative gains are added with a negative signal.

Hence, his net yardage, considering that he gained 20 yards on his last 7 carries, is given by:

6 + 20 = 26.

More can be learned about net yardage at https://brainly.com/question/14350453

#SPJ1

Help me show your work

Answers

The value of x should be greater than or equal to -2. The number line from -2 to the entire right till ∞ of the number line will satisfy this condition.

What is a number line?

A number line is just that – a straight, horizontal line with numbers placed at even increments along the length. It’s not a ruler, so the space between each number doesn’t matter, but the numbers included on the line determine how it’s meant to be used.

The value of x that will satisfy this condition can be found by simplifying the given inequality. Therefore, The given inequality can be simplified as,

4x + 1 - 1 ≥ -8

4x ≥ -8

x ≥ -2

Hence, the value of x should be greater than or equal to -2. The number line from -2 to the entire right till ∞ of the number line will satisfy this condition.

Learn more about the Number line:

https://brainly.com/question/557284

#SPJ1

Type the correct answer in each box. Use numerals instead of words. If necessary, use / for the fraction bar(s).

Answers

Answer:

3 days, 27

Step-by-step explanation:

After 3 days :

3³ = 27 belts4(3) + 15 = 12 + 15 = 27 belts

After 3 days, the number of belts and wallets sold will be the same, equal to 27.

After 3 days, the number of belts and wallets sold will be the same, equal to 27.

It is given that the number of belt sold s, x days after its online lunch  s=3^x

and s=4x+15.

To find correct answer in each box.

What is arithmetic?

The branch of mathematics dealing with the properties and manipulation of numbers.

After 3 days :

3³ = 27 belts

4(3) + 15 = 12 + 15 = 27 belts

So, after 3 days, the number of belts and wallets sold will be the same, equal to 27.

Learn more about arithmetic here:

https://brainly.com/question/27959671

#SPJ5

Triangle A C D is shown. A line is drawn from point D to point B on side A C to form a right angle. Line A D is labeled s. The length of A B is 8, the length of B C is 5, and the length of B D is 15.
What is the value of s in units?

Answers

Answer:

s = 17units

Step-by-step explanation:

For this problem, we are trying to find a specific unknown side length.

We're actually given some extraneous information (information that is not needed to solve the problem):  It isn't necessary to know that BC is 5.

If the side AD with the unknown length is part of a right triangle (the triangle in red in the attached diagram), we can use the Pythagorean Theorem to solve for AD.

It isn't clear if the diagram you were provided gives ∠ABD as a right angle,  if it only gives ∠CBD as a right angle, or if it gives both as a right angle.  Below, we prove that it doesn't matter, because regardless, both must be right angles.

Is Triangle ABD a "right triangle"?

Since B is between A and C, then the two angles ∠ABD & ∠CBD form a linear pair, and by the linear pair postulate are supplementary.  Since they are supplementary, their measures add to 180°.  Using the fact that all right angles are 90°, substitution, the subtraction property of equality, arithmetic, the measure of ∠ABD is also 90°, and thus must be a right angle.  Thus, based on the given information, both ∠ABD & ∠CBD must be right angles.

Consequently, triangle ABD is a right triangle, by definition (it is a triangle that has a right angle).

Pythagorean Theorem

Since triangle ABD is a right triangle, the Pythagorean Theorem can be applied.

The Pythagorean Theorem states that [tex]a^{2} +b^{2} =c^{2}[/tex] where "c" is the hypotenuse (the side across from the right angle) and "a" and "b" the the lengths of the two other sides (called legs) of the right triangle.  (Aside: Because of the commutative property of addition, it doesn't matter which of the two legs' lengths is used for a, and which is used for b.  The only thing that is required is that "c" be the length of the hypotenuse)

In our triangle, side AD, with unknown length "s" is the length of our hypotenuse, and sides AB and BD are the two legs.  Substituting values into the Pythagorean Theorem equation, we can solve for the unknown "s":

[tex]a^{2} +b^{2} =c^{2}[/tex]

[tex](8)^{2} +(15)^{2} =(s)^{2}[/tex]

[tex]64 +225 =s^{2}[/tex]

[tex]289 =s^{2}[/tex]

Applying the square root property...

[tex]\pm \sqrt{289} =\sqrt{s^{2}}[/tex]

[tex]s=17 \text{ or } s=-17[/tex]

Final Solution

We discard the negative solution we obtained, since s represents the length of the side of a triangle.

s = 17units

Answer:

17

Step-by-step explanation:

in an isosceles triangle, the perimeter is 75 cm and one of the sides is 25 cm. Find all its sides. can you find all angles of the triangles?

Answers

Answer:

all angles of the triangles is 60

Step-by-step explanation:

the perimeter = 2*side of an isosceles triangle + bottom edge

----> bottom edge = the perimeter - 2*side of an isosceles triangle  =25

----> This is an equilateral triangle ----> all angles of the triangles is 60

(-3 +6 i )(-3 - 6 i )

Answers

Answer:

45

Step-by-step explanation:

What is the range of the function y=-x² +1?
A) y≤ -1
B) y²-1
C) y≤ 1
D) y≥ 1

Answers

Answer:

Option (C)

Step-by-step explanation:

The minimum value of x² is 0, and the maximum value is unbounded, so therefore, the maximum value of -x² is 0, and the minimum value is unbounded.

So, this means that adding 1 to this, the range matches with option C.

What is the quotient of the synthetic division problem below, written in
polynomial form?
3 -2 15
-22
-16
A.2xd 121xr 41
B=
2,219x15
C. 2y 1 9g
40
D
2y7 - 21x 40

Answers

The quotient of the synthetic division problem is "f(x) = -2x² + 9x + 5" (Option C).

What is Synthetic Division?

When a polynomial division is executed using the shorthand method, using a linear factor, one is  said to have executed a Synthetic Division.

The synthetic division translates to

- 2 15 -22 -15

3 | -6 27 15

-2 9 5 0

Citing the coefficient of the quotient of the first three numbers:

- 2, 9, 5. The last number is given as the remainder.

Hence the remainder is zero.

Given that the coefficient of the quotient are three, the interpretation is that the polynomial greatest degree is 2.

Thus,

f(x) = -2x² + 9x + 5

Learn more about synthetic division at:
https://brainly.com/question/18788426
#SPJ1

Zippy company manufactured 10,000 units in december with a total product cost of $22,900 they had zero finished goods inventory at the start of december. in december zippy sold 7,796 units at a unit price of $5.54. period expenses were $4,723. what is the amount of zippy's gross profit (otherwise know as gross margin).? round any intermediate calculations to four decimal places

Answers

The amount of zippy's gross profit (otherwise know as gross margin) is: $25,337.

Gross profit

Sales revenue $43,189.84

(7,796 units × $5.54)

Less Cost of goods sold $17,852.84

[($22,900/10,000)×7,796]

Gross profit $25,337

( $43,189.84-$17,852.84)

Therefore the amount of zippy's gross profit (otherwise know as gross margin) is: $25,337.

Learn more about gross profit here:https://brainly.com/question/942181

#SPJ1

Janet solves this equation

log(x-3)+logx=1

She finds the two solutions; x=5 and x=-2

Of Janet’s two solutions, ____ correct because _____

A. Neither x=5 nor x=-2 is
B. Only x=5 is
C. Only x=-2 is
D. Both x=5 and x=-2 are

1. x=5 is an extraneous solution
2. Both x=-2 and x=5 are valid solutions
3. Both x=-2 and x=5 are extraneous solutions
4. x=-2 is an extraneous solution

(TWO DIFFERENT ANSWERS FILL IN THE BLANK)

Answers

Of Janet’s two solutions, both x=5 and x=-2 are correct because both x=-2 and x=5 are extraneous solutions

Logarithmic function

Given the log function expressed as:

log(x-3) + logx=1

According to the law of logarithm, addition becomes product to have:

log x(x -3) = log₁₀10

x² - 3x = 10

x² - 3x -10 = 0
x² - 5x + 2x - 10 = 0
x(x-5) + 2(x-5) = 0
(x+2)(x-5)=0

x = -2 and 5

Of Janet’s two solutions, both x=5 and x=-2 are correct because both x=-2 and x=5 are extraneous solutions

Learn more on logarithm here: https://brainly.com/question/25710806

#SPJ1

Separating variable in the equation gives you the following equation:​

Answers

Answer:

[tex]\textsf{E.} \quad y\:dy=xe^x\:dx[/tex]

Step-by-step explanation:

Given equation:

[tex]ye^{-x}\dfrac{dy}{dx}=x[/tex]

Divide both sides by [tex]e^{-x}[/tex]:

[tex]\implies \dfrac{ye^{-x}}{e^{-x}}\:\dfrac{dy}{dx}=\dfrac{x}{e^{-x}}[/tex]

[tex]\implies y\:\dfrac{dy}{dx}=\dfrac{x}{e^{-x}}[/tex]

Multiply both sides by [tex]dx[/tex] :

[tex]\implies y\:\dfrac{dy}{dx}\cdot dx=\dfrac{x}{e^{-x}}\cdot dx[/tex]

[tex]\implies y\:dy=\dfrac{x}{e^{-x}}\:dx[/tex]

[tex]\textsf{Apply exponent rule} \quad \dfrac{1}{a^{-n}}=a^n:[/tex]

[tex]\implies y\:dy=xe^x\:dx[/tex]

Answer: E

Step-by-step explanation:

[tex]ye^{-x} \frac{dy}{dx}=x\\\\y \frac{dy}{dx}=xe^{x} \\ \\ \boxed{y dy=xe^{x} dx}[/tex]

All of the details to the question are in the picture that is attached in this question, please help

Answers

Answer: 10

The arc length of the semicircle is

a+(a+1)+(a+2)+(a+3)+(a+4)=5a+10

As x = 42, this means a+4 is 42/180 of the arc length of the circle, 5a+10.

So,

(42/180)(5a+10)=a+4

42(5a+10)=180(a+4) [multiply both sides by 180]

210a+420=180a+720 [distributive property]

30a+420=720 [subtract 180a from both sides]

30a=300 [subtract 420 from both sides]

a=10 [divide both sides by 30]

Emily works at a local restaurant. She made $200 in tips last night. She shared 15 percent of her tips with the crew that cleans the tables. How much did she give to the clean-up crew?

$

Answers

A percentage is a way to describe a part of a whole. The amount Emily shared is $30.

What are percentages?

A percentage is a way to describe a part of a whole. such as the fraction ¼ can be described as 0.25 which is equal to 25%.

To convert a fraction to a percentage, convert the fraction to decimal form and then multiply by 100 with the '%' symbol.

Emily works at a local restaurant. She made $200 in tips last night. She shared 15% of her tips with the crew that cleans the tables. The amount Emily shared is,

15% of $200

= 0.15 × $200

= $30

The amount Emily shared is $30.

Learn more about Percentages:

https://brainly.com/question/6972121

#SPJ1

Triangle ABC is shown on the graph. What are the coordinates of the image of point B after the triangle is rotated 270° about the origin?
(4, 2)
(2, 4)
(–4, –2)
(–2, –4)

Answers

The coordinates of the image of point B after the triangle is rotated 270° about the origin is (4, 2)

How to determine the image of point B?

The complete question is added as an attachment

From the attached image, we have the following coordinate

B = (-2, 4)

When the triangle is rotated by 270 degrees, the rule of rotation is:

(x, y) ⇒ (y, -x)

For point B, we have:

B' = (4, 2)

Hence, the coordinates of the image of point B after the triangle is rotated 270° about the origin is (4, 2)

Read more about rotation at:

https://brainly.com/question/7437053

#SPJ1

How do you use the additive inverse to evaluate an expression that uses subtraction ?

Answers

To use the additive inverse to evaluate an expression that uses subtraction, change the sign of the number to positive

What is additive inverse?

Additive inverse is simply changing the sign of a number and then adding it to the original number to get an answer that is equal to 0.

The additive inverse of a  number is another number

The additive inverse of number 3 is - 3

For an expression that uses subtraction, to use the additive inverse, change the sign of the number to positive

Thus, to use the additive inverse to evaluate an expression that uses subtraction, change the sign of the number to positive

Learn more about additive inverse here:

https://brainly.in/question/15948306

#SPJ1

Domain of y=5x+2 help!!!!

Answers

the domain is y’=5.

Please help asap thanks so much!

Answers

Answer:

see the attachment photo!

When planning road development, the road commission estimates the future population using the function represented in the table, where x is the time in years and f(x) is the total population.



What is the significance of 160,000 in the function?

the maximum population of the city
the expected population in 5 years
the initial population at the time of the estimation
the amount of increase in the popu

Answers

Considering that 160,000 is the y-intercept of the function, it's significance is given by:

the initial population at the time of the estimation.

What is the y-intercept of a function f(x)?

The y-intercept is f(0), that is, the value of y when x = 0, which is interpreted as the initial value of the function.

Researching this problem on the internet, it is found that f(0) = 160,000, hence the significance of 160,000 in the function is given by:

the initial population at the time of the estimation.

More can be learned about the y-intercept of a function at https://brainly.com/question/27979095

#SPJ1

Round 2
5. A Cadillac Escalade gasoline tank has a capacity of 24
gallons. The car uses approximately 1 gallon for every 25
miles it drives. If the tank starts full, write an equation that
describes the amount of gas, g, left in the tank after it has
been driven for m miles.

Answers

Answer:

g = 24 - 1/25m

Step-by-step explanation:

Since the Cadillac Escalade can travel 25 miles per gallon, each mile the Escalade travels is 1/25th of a gallon of gas.

This means that every mile the Escalade travels, 1/25th of a gallon is subtracted from the gas tank. Since the gas tank has a capacity of 24 gallons, the value of the gas tank is 24.

This leads us to the equation:

g = 24 - 1/25m

where g is the amount of gas left, and m is the number of miles driven.

Determine the linear function for a graph that is a line with a slope of 1/7
and contains the point
(−4, 1).

Answers

The linear equation is y = (1/7)*x + 11/7.

How to get the linear equation?

The general linear equation is:

y = a*x + b

Where a is the slope.

Here we know that a = (1/7), then the line is:

y = (1/7)*x + b

We also know that the line contains the point (-4, 1), this means that when x = -4, we must have y = 1.

Replacing that, we get:

1 = (1/7)*(-4) + b

1 + 4/7 = b

7/7 + 4/7 = b

11/7 = b

So the linear equation is:

y = (1/7)*x + 11/7.

If you want to learn more about linear equations:

https://brainly.com/question/1884491

#SPJ1

The equation of the linear function of the line is: y = 1/7x + 11/7.

What is a Linear Function Equation?

A linear function equation is modelled as, y = mx + b, where m is the slope and b is the y-intercept.

Slope (m) = 1/7, and the line passes through (-4, 1), therefore, substitute (x, y) = (-4, 1) and m = 1/7 into y = mx + b:

1 = 1/7(-4) + b

1 = -4/7 + b

1 + 4/7 = b

11/7 = b

b = 11/7

Plug in the values of m and b into y = mx + b:

y = 1/7x + 11/7

The equation of the linear function is: y = 1/7x + 11/7

Learn more about linear function on:

https://brainly.com/question/1362601

#SPJ1

√2/√2+√3-√5 rationalise the denominator

Answers

by rationalising
answer is
(2√6+√15+6)/12

Match each pair of equivalent values.

1 .
1
square root of 16
2 .
12
square of 8
3 .
4
4 2
4 .
64
1 2
5 .
16

6 .
2

Answers

Answer:

1 - [tex]1^{2}[/tex]

12 - ?

4 - square root of 16

64 - [tex]8^{2}[/tex]

16 - [tex]4^{2}[/tex]

2.828 - square root of 8

Step-by-step explanation:

I need more terms to match

Square root of 16 is 4, square root of 8 is 2, but for an answer it would be , 4² is 16, 1² is 2, number 5 would be 64 and number 6 would be 12.

What are mathematical values?

A mathematical value can generally be any specific mathematical object. This is most frequently a number in elementary mathematics, such as a real number like or an integer like 42. Every integer or even other mathematical object set to a variable or constant is that object's value.

A mathematical expression's value is the outcome of a computation it describes when its variables and constants are given values. The amount that the function assumes for these argument numbers is the result of a function, assuming the value(s) provided to its argument(s).

Square root of 16 is 4, square root of 8 is 2, but for an answer it would be , 4² is 16, 1² is 2, number 5 would be 64 and number 6 would be 12.

Therefore, Square root of 16 is 4, square root of 8 is 2, but for an answer it would be , 4² is 16, 1² is 2, number 5 would be 64 and number 6 would be 12.

To know more about mathematical values, here:

brainly.com/question/30402650

#SPJ2

please place the dot for me(20 points will give brainliest!!!)

Answers

Answer:

Domain & Range are both all real numbers.

Step-by-step explanation:

i

If this was a traditional parabola, the range would be y<=4. However, the arrow on the left going back up means eventually y will go to both positive & negative infinity.

Step 1: We know that Angle A B C Is-congruent-to Angle F G H because all right angles are congruent. Step 2: We know that Angle B A C Is-congruent-to Angle G F H because corresponding angles of parallel lines are congruent. Step 3: We know that Line segment B C is-congruent-to line segment G H because it is given. Step 4: Triangle A B C Is-congruent-to Triangle F G H because of the

Answers

Triangles FGH and ABC are congruent because of the: AAS congruence theorem.

What is the AAS Congruence Theorem?

The AAS congruence theorem states that when two angles and one non-included side in one triangle are congruent to corresponding two angles and one non-included side in another triangle, then both triangles are congruent.

In the proof given, it is established that both triangles have two corresponding congruent angles, and also, BC ≅ GH which are non-included sides.

Therefore, both triangles are congruent because of the AAS congruence theorem.

Learn more about the AAS congruence theorem on:

https://brainly.com/question/3168048

#SPJ1

This table represents a quadratic function with a vertex at (1, 1). What is the
average rate of change for the interval from x = 5 to x = 6?
OA. 26
OB. 13
O C. 7
OD. 9
1
2
3
4
5
X
1
2
LO
5
10
17
y

Answers

The average rate of change on the interval (5, 6) is 9. So the correct option is D.

How to find the average rate of change on the interval?

Here we want to find the average rate of change of f(x), the function on the table, on the interval (5, 6).

This is just:

[tex]r = \frac{f(6) - f(5)}{6 - 5}[/tex]

[tex]f(x) = a*x^2 + b*x + c[/tex]

I we look at the table we see that:

[tex]f(1) = 1 = a + b + c[/tex]

[tex]f(2) =2 = 4a + 2b + c[/tex]

[tex]f(3) = 5 = 9a + 3b + c[/tex]

This is a system of equations.

If we subtract the second and first functions, we get:

[tex]2 - 1 = (4a + 2b +c) - (a + b + c)\\1 = 3a + b = a + b + c[/tex]

From that we take two relations:

[tex]1 - 3a = b\\2a = c[/tex]

Now we can replace these two in the last equations so we get:

[tex]5 = 9a + 3b + c\\\\5 = 9a + 3*(1 - 3a) + 2a\\\\5 = 9a + 3 - 9a + 2a\\\\5 = 3 + 2a\\\\5 - 3 = 2a\\\\2 = 2a\\\\a = 1[/tex]

Now that we know the value of a:

[tex]c = 2a = 2*1 = 2\\\\b = 1 - 3a = 1 - 3 = -2[/tex]

The quadratic equation is:

[tex]f(x) = x^2 - 2x + 2[/tex]

Evaluating this in x = 6 we get:

[tex]f(6) = 6^2 - 2*6 + 2 = 26[/tex]

And from the table we know that f(5) = 17, then the average rate of change is:

[tex]r = \frac{f(6) - f(5)}{6 - 5} = \frac{26-17}{1} = 9[/tex]

The correct option is D.

If you want to learn more about average rates of change:

https://brainly.com/question/8728504

#SPJ1

4
12 + 17 < -18 *
True
False

Answers

did you mean 4 multiply by -18 on your right side? if so, it would be:

12+ 17 = 29
4 * -18 = -72

so the answer would be false, since 29 is greater than -72
False! Because -18 is definitely not greater

Select the correct answer. A linear function on a coordinate plane passes through (minus 3, 2), (0, 4), and (3, 6) Which equation describes the line graphed above? A. B. C. D.

Answers

Answer:

I don't see the equation options.  See below for a predicted equation.

y= (2/3)x + 4

Step-by-step explanation:

The points are (-3,2), (0,4), and (3,6).  I will assume they lie in a straight line.

Find an equation of the form y=mx+b, where m is the slope and b is the y-intercept (the value of y when x=0).

m, the slope, is also known as the "Rise/Run."

Pick any two points.  I'll use (0,4) and 3,6)

Rise = (6-4) = 2

Run  = (3-0) = 3

Slope = 2/3

The equation becomes y = (2/3)x + b

B is easy in this case.  Point (0,4) tells us that y = 4 when x = 0 (the definition of b).

The equation is y= (2/3)x + 4

See the attached graph.

Select all the correct answers.
Which expressions are equivalent to this exponential expression? Please help

Answers

An expression is defined as a set of numbers, variables, and mathematical operations. The correct options are A and C.

What is an Expression?

In mathematics, an expression is defined as a set of numbers, variables, and mathematical operations formed according to rules dependent on the context.

The given exponential (6⁻¹⁰/6⁻⁴) function can be simplified as shown below,

(6⁻¹⁰/6⁻⁴)

= 6⁻¹⁰ × 6⁴

= 6⁽⁻¹⁰⁺⁴⁾

= 6⁻⁶

= 1/6⁶

Hence, the correct options are A and C.

Learn more about Expression:

https://brainly.com/question/13947055

#SPJ1

Other Questions
The surface area of the cone below is about 151.58 inches squared. The radius of the base is 4 inches. What is the slant height? Use 3.14 for Pi. Round your answer to the nearest whole number. Under which circumstance may the maximum person capacity of a vessel be exceeded?. A type of party formation in which political conflict prompts officials and competing factions within government to mobilize popular support is called __________ mobilization. 15) You areencouraged to manageyour debt by:A) Onlygoing intodebt forthings youreally needand haveplanned for.B) Makingregular,agreed-upon debtrepayments.C) Gettingone bigbank loan topay all youroutstandingdebt. Doubling the dimensions of a rectangle increases the area by a factor of 4.If p represents doubling the dimensions of a rectangle and q represents the area increasing by a factor of 4, which are true? Select two options. _____ is a model in which application service providers (asps) deliver software to users for a fee Describe how basidiospore is produced Suzannas doctor told her to lose 9 kg, which was 10% of her body weight.a) How much did Suzanna weigh?b) If Suzanna lost 12% of her body weight, what is her new weight? Set up a proportion to solve for x in the following similar triangles it's okay because Kenny G is always the right answer On a zip-line course, you are harnessed to a cable that travels through the treetops. You start at Platform A and zip to each of the other platforms. How far do you travel from Platform B to Platform C? Write your answer to the nearest tenth of a foot. A green gold bracelet weighs 56g. The bracelet is made from 75% gold, 20% silver and 5% copper.(a) What is the mass of the gold in the bracelet?(b) What is the mass of the copper in the bracelet? Chocolate cost $1.50 and was paid for with $5 which three coins were given as change A shipment of tile cost $64,500. each tile costs 15 dollars. how many pieces of tile are in the shipment? Identify figure of speech"An old man is the wisest fool"hyperboleoxymoronmetaphorsimile 1.Which thematic group uses technology to direct the behavior of dynamical systems, ensuring that they behave in a predictable manner?2.Which thematic group creates circuits with miniature, packaged components that directs and control electricity?a.Communication Systemsb.Control Systemsc.Computer Systemsd.Energy Systemse.Electronic Systems Immigrants were targeted during the Palmer Raids because they wereA. entering the United States illegally and taking low-wage jobsB. gathering in large ethnic communities, harming property valuesC. aggressively trying to overthrow the US governmentD. believed to be sympathetic to anarchists and communists [20 points + brainliest]How much do you think place affects people? Consider how a certainplace has played a part in your life or in the life of someone you know. You could also think about a movie or a book in which the location is key to the story. Write a paragraph explaining how you think place affects a person or character. Read the excerpt from Eleanor Roosevelts speech "The Struggle for Human Rights.As I see it, it is not going to be easy to attain unanimity with respect to our different concepts of government and human rights. The struggle is bound to be difficult and one in which we must be firm but patient. If we adhere faithfully to our principles I think it is possible for us to maintain freedom and to do so peacefully and without recourse to force.The future must see the broadening of human rights throughout the world. People who have glimpsed freedom will never be content until they have secured it for themselves. In a truest sense, human rights are a fundamental object of law and government in a just society. Human rights exist to the degree that they are respected by people in relations with each other and by governments in relations with their citizens.What type of evidence does Roosevelt use to support the claim that the United Nations must attain unanimity on the human rights issue? Paulo is flying a kite as shownin the right. Find the length of the kite string.